药理学英文试题(含答案) 下载本文

内容发布更新时间 : 2024/5/5 6:43:54星期一 下面是文章的全部内容请认真阅读。

1山东大学医学院(2005 —— 2006 学年第二学期)

2003 级 医学七年制 药理学期末考试试题(A 卷)

二、选择题:( A 型每题 0.5 分,X 型每题 1分, 共 50 分)

Type A choice questions (only one answer is correct)

1. Which of the following is classified as belong to the G protein coupling receptors? A. GABAA receptor B. badrenergic receptor

C. insulin receptor D. nicotinic II receptor E. hydrocortisone receptor

2. Which of the following is unlikely to be associated with oral drug administration of an

entericcoated dosage form?

A. irritation to the gastric mucosa with nausea and vomiting

B. destruction of the drug by gastric acid or digestive enzymes

C. unpleasant taste of the drug

D. formation of nonabsorbable drugfood complexes

E. variability in absorption caused by fluctuations in gastric emptying time 3. Which of the following compounds will be absorbed to the least extent in the stomach?

A. ampicillin (pKa=2.5) B. aspirin (pKa=3.0) C. warfarin (pKa=5.0)

D. Phenobarbital (pKa=7.4) E. propranolol (pKa=9.4)

4. Which of the following terms is most likely to be associated with “a rapid reduction in the

effect of a given dose of a drug after only one or two doses”?

A. supersensitivity B. tachyphylaxis C. tolerance

D. hyposensitivity E. anaphylaxis

5. A weak acidic drug with 4.4 of pKa, if the stomach juice pH is 1.4, plasma pH is 7.4, when

the distribution balance is reached, the drug concentration ratio between plasma and stomach juice is A. 100 B. 0.001 C. 1000 D. 10000 E. 0.01

6. In this graph,drugs A,B and C are analogs,then

A. A has a greater potency than B B. A has a greater efficacy than C C. B has less potency than C

D. A has a greater potency than C E. B has a greater efficacy than C

7. Aspirin is a weak acid with 3.5 of pKa, what percentage of lipidsoluble form will be in the

stomach juice with 2.5 of pH? A. 0.99% B. 9% C. 9.09% D. 90.9% E. 99.9%

8. The contractile effect of various doses of norepinephrine (NE) (X) alone on vascular smooth

muscle is represented in the figure below. When combined with an antagonist (IC or INC), a

shift in the doseresponse

curve occurs. The curve labeled X+INC would mostly likely occur

when vascular smooth muscle is treated with NE in the presen

A. terazosin B. phentolamine C. labetalol

D. phenoxybenzamine E. prazosin B A C 效 应

Log 剂量 4

9. The reversible cholinesterase inhibitor indicated in the treatment of Alzheimer’s disease is A. tacrine

B. edrophonium C. neostigmine D. pyridostigmine E. ambenonium

10. A predictably dangerous side effect of nadolol that constitutes a contraindication to its

clinical use in susceptible patients is the induction of A. hypertension

B. cardiac arrhythmia C. asthmatic attacks

D. respiratory depression E. hypersensitivity

11. Epinephrine may be mixed with certain anesthetics, such as procaine, in order to A. stimulate local wound repair B. promote hemostasis

C. enhance their interaction with neural membranes and their ability to depress nerve

conduction

D. retard their systemic absorption

E. facilitate their distribution along nerves 12. A 58yearold

male with angina is treated with atenolol. Select the mechanism of action of atenolol A. aadrenergic agonist

B. aadrenergic antagonist C. badrenergic agonist

D. badrenergic antagonist

E. mixed a and b antagonist

13. A male patient is brought to the

emergency department following ingestion of an unknown

substance. He is found to have an elevated temperature, hot and flushed skin, dilated pupils,

and tachycardia. Of the following, which would most likely cause these findings? 5

A. propranolol B. tolazoline C. prazosin D. donepezil E. atropine 14. A 65yearold

male has a blood pressure of 170/105mmHg. Which of the following would

be effective in lowering this patient’s blood pressure? A. terbutaline B. dobutamine C. pancuronium D. prazosin E. scopolamine

15. Which of the following can be used in shock for increasing cardiac output and renal blood flow?

A. norepinephrine B. epinephrine C. dopamine D. phenylephrine E. methoxamine

16. All of the following statements are related with succinylcholine EXCEPT

A. have muscle fasciculation before muscle relaxation

B. have no ganglionic blocking actions at therapeutic dose

C. can be antagonized by neostigmine D. elevate blood potassium concentration E. assistant agents of anesthetics 17. The agent which is effective for

prostatic hyperplasia but has no effect on blood pressure is A. terazosin B. tamsulosin

C. phenoxybenzamine D. regitine E. prazosin

18. Which of the following statements is correct?

A. Diazepam induce the drug metabolizing enzymes.

B. All benzodiazepines show antiepileptic actions.

C. All benzodiazepines have sedative effects.

D. Benzodiazepines readily produce general anesthesia.

E. Benzodiazepines directly open chloride channels.

19. Which of the following statements is correct?

A. Chlorpromazine is indicated in treating the nausea of levodopa treatment.

B. Vitamin B6 increases the effectiveness of levodopa. 6

C. Administration of dopamine is an

effective treatment of Parkinson’s disease. D. Levodopainduced

nausea is reduced by carbidopa. E. Nonspecific MAOinhibitors

are a useful adjunct to levodopa therapy. 20. Which of the following is common to the tricyclic antidepressants and MAO inhibitors?

A. They can produce sedation. B. They produce physical dependence. C. They show strong interaction with certain foods.

D. They can produce postural hypotension. E. They decrease availability of epinephrine in the synaptic cleft

21. The antipsychotic drugs:

A. are equally effective against the positive and negative symptoms of schizophrenia B. can cause blurred vision, urinary retention and other signs of muscarinic blockade

C. bind selectively to D2dopaminergic receptors.

D. have antiparkinsonism effects similar to levodopa.

E. have a rapid onset of antipsychotic action.

22. All of the following are observed in patients taking neuroleptic agents EXCEPT:

A. sexual dysfunction. B. bronchial asthma

C. altered endocrine function. D. constipation.

E. orthostatic hypotension

23. Which of the following statements about morphine is INCORRECT?

A. It is used therapeutically to relieve pain caused by severe head injury.

B. Its withdrawal symptoms can be relieved by methadone.

C. It causes constipation.

D. It is most effective by parenteral administration.

E. It rapidly enters many body tissues, including the fetus of a pregnant woman. 24. Which of the following statements concerning phenytoin is INCORRECT? A. causes less sedation than phenobarbital. B. causes gingival hyperplasia.

C. may cause megaloblastic anemia D. is excreted unchanged in the urine. E. The plasma halflife

increases as the dose is increased. 25. Which of the following drugs is contraindicated in the patients with epilepsy?

A. Phenobarbital B. imipramine C. digoxin

D. chlorpromazine E. aspirin

26. The agent which is effective for various epilepsy is 7

A. diazepam

B. sodium phenytoin C. sodium valproate D. Phenobarbital E. ethosuximide

27. All of the following produce a

significant decrease in peripheral resistance EXCEPT:

A. chronic administration of diuretics B. hydralazine C. ACE inhibitors D. βR blockers

E. calcium channel blockers

28. Which of the following hypertensive patients is most suitble for primary therapy with

hydrochlorothiazide? A. patients with gout

B. patients with hyperlipidemia

C. young hypertensive patients with rapid resting heart rate

D. patients with impaired renal function E. elderly patients

29. Regarding antihypertensive drugs, which of the following statements is WRONG?

A. Hydrochlorothiazide can increase activity of rennin

B. Propranolol can decrease secretion of rennin

C. Sodium nitroprusside lowers BP rapidly by releasing NO.

D. Clonidine stimulates α2receptor and imidazoline receptor

E. Diuretics decreased BP mainly by increasing water and sodium excretion from the kidneys.

30. Which of the following drugs occurs orthostatic hypotension most frequently in first use: A. clonidine B. nifedipine C. propranolol D. enalapril E. prazosin

31. The reason that digoxin can reduce the ventricular rate of atrial fibrillation patient is:

A. decreasing automaticity of ventricles B. decreasing automaticity of atria C. reducing the conduction of AV node

D. improving cardiac ischemia

E. shortening the effective refractory period of atria

32. All of the following measures can be used in the treatment of digoxininduced arrhythmia EXCEPT

A. stopping digoxin administration 8

B. diuretic agents such as furosemide are used to promote the excretion of digoxin C. phenytoin administration D. atropine administration E. lidocaine administration

33. Which of the following effects of digoxin can NOT be seen in failure heart? A. slowing sinus rhythm

B. increasing the oxygen consumption of myocardia

C. increasing the cardiac output

D. increasing the cardiac contractility E. shortening atrial ERP

34. A 70 year old

female is treated with sublingual

nitroglycerin for her occasional bouts of angina. Which of the following is involved in the action of nitroglycerin? A. αadrenergic activity

B. phosphodiesterase activity

C. phosphorylation of light chains of myosin

D. norepinephrine release E. cGMP increased

35. The therapeutic effect of β adrenergic receptor blockers such as propranolol in angina

pectoris is believed to be primarily the result of

A. reduced production of catecholamines B. dilation of the coronary vasculature C. decreased requirement for myocardial oxygen

D. increased peripheral resistance

E. increased sensitivity to catecholamines 36. A 69yearold

male with angina develops severe constipation following treatment with A. nitroglycerin

B. gemfibrozil(吉非贝齐) C. propranolol D. captopril E. verapamil

37. Which of the following drugs has

relatively few electrophysiologic effects on normal

myocardial tissue but suppresses the arrhythmogenic tendencies of ischemic myocardial tissue?

A. Verapamil B. Lidocaine C. Quinidine D. Propranolol E. Procainamide 38. The firstline

drug for treating acute attack of reentrant supraventricular tachycardia is A. adenosine 9

B. lidocaine C. quinidine D. digoxin

E. procainamide

39. Which of the following drugs is NOT suitable to treat the patient with angina pectoris

accompanied with asthma? A. nifedipine

B. isosorbide dinitrate C. nitroglycerin D. verapamil E. propranolol

40. Which of the following drugs decreases de novo cholesterol biosynthesis by inhibiting the

enzyme HMG CoA reductase? A. nicotinic acid B. gemfibrozil C. lovastatin

D. cholestyramine(考来烯胺) E. probucol

41.Which of the following drugs can be used for Diabetics(typeⅡ)with renal insufficiency A. gliclazide B. tolbutamide C. glibornuride D. gliquidone E. glipizide

42. The contraindication of Glucocorticoids is

A. septicemia B. iritis

C. dermatomyositis

D. epidemic parotitis(流行性腮腺炎) E. peptic ulcer

43. Antiasthmatic that could inhibit

leukotrienes release and increase cerebral blood flow is